How to Maximize the Volume of a Cone Inside a Sphere?

Click For Summary
To find the volume of the largest right circular cone inscribed in a sphere of radius 3, the volume formula V = πr^2h/3 is used, with r and h defined in relation to the sphere's dimensions. The user attempted to derive the volume with h expressed as (9-x^2)^(1/2) but is uncertain about the correctness of their derivative. Another participant suggests expressing V in terms of x and indicates they obtained a different derivative, prompting further clarification on the steps involved. The discussion highlights the need for a clearer approach to derive the volume function and optimize it. Overall, the conversation centers on solving the optimization problem for the cone's volume within the sphere.
Willowz
Messages
197
Reaction score
1

Homework Statement


Find the volume of the largest right circular cone that can be inscribed in a sphere of radius 3.

Homework Equations


V=π*r^2*h/3
A=πr^2 + πrl

The Attempt at a Solution


I did multiple things that I'm not sure are correct. I took the derivative for the volume with the value of h set to (9-x^2)^(1/2). The derivative I got was;
3π/2 * (9-x^2)^(-1/2) * (-2x)

Not really sure what to do. Think I need a hint on the steps that need to be taken. Picture of problem attached.
 

Attachments

  • problem.jpg
    problem.jpg
    17.1 KB · Views: 466
Last edited:
Physics news on Phys.org
In your diagram, I would replace "y" with "h - 3". Next, find x in terms of h. Then substitute into the volume of a right circular cone
V = \frac{1}{3}\pi r^2 h
(with r = x here) and you'll have a function with one variable, h. Now find dV/dh and go on from there.
 
Willowz said:

Homework Statement


Find the volume of the largest right circular cone that can be inscribed in a sphere of radius 3.


Homework Equations


V=π*r^2*h/3
A=πr^2 + πrl

The Attempt at a Solution


I did multiple things that I'm not sure are correct. I took the derivative for the volume with the value of h set to (9-x^2)^(1/2). The derivative I got was;
3π/2 * (9-x^2)^(-1/2) * (-2x)

Not really sure what to do. Think I need a hint on the steps that need to be taken. Picture of problem attached.

Write out your expression for V in terms of x. I got a different derivative dV/dx than yours.

RGV
 
Question: A clock's minute hand has length 4 and its hour hand has length 3. What is the distance between the tips at the moment when it is increasing most rapidly?(Putnam Exam Question) Answer: Making assumption that both the hands moves at constant angular velocities, the answer is ## \sqrt{7} .## But don't you think this assumption is somewhat doubtful and wrong?

Similar threads

Replies
9
Views
4K
  • · Replies 4 ·
Replies
4
Views
2K
Replies
5
Views
2K
Replies
3
Views
2K
  • · Replies 17 ·
Replies
17
Views
5K
  • · Replies 13 ·
Replies
13
Views
3K
  • · Replies 1 ·
Replies
1
Views
2K
  • · Replies 21 ·
Replies
21
Views
3K
  • · Replies 3 ·
Replies
3
Views
2K
  • · Replies 5 ·
Replies
5
Views
6K